summaryrefslogtreecommitdiff
path: root/Principles_And_Modern_Applications_Of_Mass_Transfer_Operations/chapter8.ipynb
blob: 8b6f4f06838aeb4e99d99a72f6d48fb523cca90f (plain)
1
2
3
4
5
6
7
8
9
10
11
12
13
14
15
16
17
18
19
20
21
22
23
24
25
26
27
28
29
30
31
32
33
34
35
36
37
38
39
40
41
42
43
44
45
46
47
48
49
50
51
52
53
54
55
56
57
58
59
60
61
62
63
64
65
66
67
68
69
70
71
72
73
74
75
76
77
78
79
80
81
82
83
84
85
86
87
88
89
90
91
92
93
94
95
96
97
98
99
100
101
102
103
104
105
106
107
108
109
110
111
112
113
114
115
116
117
118
119
120
121
122
123
124
125
126
127
128
129
130
131
132
133
134
135
136
137
138
139
140
141
142
143
144
145
146
147
148
149
150
151
152
153
154
155
156
157
158
159
160
161
162
163
164
165
166
167
168
169
170
171
172
173
174
175
176
177
178
179
180
181
182
183
184
185
186
187
188
189
190
191
192
193
194
195
196
197
198
199
200
201
202
203
204
205
206
207
208
209
210
211
212
213
214
215
216
217
218
219
220
221
222
223
224
225
226
227
228
229
230
231
232
233
234
235
236
237
238
239
240
241
242
243
244
245
246
247
248
249
250
251
252
253
254
255
256
257
258
259
260
261
262
263
264
265
266
267
268
269
270
271
272
273
274
275
276
277
278
279
280
281
282
283
284
285
286
287
288
289
290
291
292
293
294
295
296
297
298
299
300
301
302
303
304
305
306
307
308
309
310
311
312
313
314
315
316
317
318
319
320
321
322
323
324
325
326
327
328
329
330
331
332
333
334
335
336
337
338
339
340
341
342
343
344
345
346
347
348
349
350
351
352
353
354
355
356
357
358
359
360
361
362
363
364
365
366
367
368
369
370
371
372
373
374
375
376
377
378
379
380
381
382
383
384
385
386
387
388
389
390
391
392
393
394
395
396
397
398
399
400
401
402
403
404
405
406
407
408
409
410
411
412
413
414
415
416
417
418
419
420
421
422
423
424
425
426
427
428
429
430
431
432
433
434
435
436
437
438
439
440
441
442
443
444
445
446
447
448
449
450
451
452
453
454
455
456
457
458
459
460
461
462
463
464
465
466
467
468
469
470
471
472
473
474
475
476
477
478
479
480
481
482
483
484
485
486
487
488
489
490
491
492
493
494
495
496
497
498
499
500
501
502
503
504
505
506
507
508
509
510
511
512
513
514
515
516
517
518
519
520
521
522
523
524
525
526
527
528
529
530
531
532
533
534
535
536
537
538
539
540
541
542
543
544
545
546
547
548
549
550
551
552
553
554
555
556
557
558
559
560
561
562
563
564
565
566
567
568
569
570
571
572
573
574
575
576
577
578
579
580
581
582
583
584
585
586
587
588
589
590
591
592
593
594
595
596
597
598
599
600
601
602
603
604
605
606
607
608
609
610
611
612
613
614
615
616
617
618
619
620
621
622
623
624
625
626
627
628
629
630
631
632
633
634
635
636
637
638
639
640
641
642
643
644
645
646
647
648
649
650
651
652
653
654
655
656
657
658
659
660
661
662
663
664
665
666
667
668
669
670
671
672
673
674
675
676
677
678
679
680
681
682
683
684
685
686
687
688
689
690
691
692
693
694
695
696
697
698
699
700
701
702
703
704
705
706
707
708
709
710
711
712
713
714
715
716
717
718
719
720
721
722
723
724
725
726
727
728
729
730
731
732
733
734
735
736
737
738
739
740
741
742
743
744
745
746
747
748
{
 "metadata": {
  "name": ""
 },
 "nbformat": 3,
 "nbformat_minor": 0,
 "worksheets": [
  {
   "cells": [
    {
     "cell_type": "heading",
     "level": 1,
     "metadata": {},
     "source": [
      "Chapter 8: Humidification Operations "
     ]
    },
    {
     "cell_type": "heading",
     "level": 3,
     "metadata": {},
     "source": [
      "Example 8.1,Page number:479"
     ]
    },
    {
     "cell_type": "code",
     "collapsed": false,
     "input": [
      "\n",
      "#Variable declaration\n",
      "\n",
      "P_total = 1  \t\t\t\t\t\t# [bar]\n",
      "T1 = 320.0  \t\t\t\t\t\t# [K]\n",
      "T_c = 562.2  \t\t\t\t\t\t# [K]\n",
      "P_c = 48.9  \t\t\t\t\t\t# [bar]\n",
      "A = -6.983 \n",
      "B = 1.332 \n",
      "C = -2.629 \n",
      "D = -3.333 \n",
      "import math\n",
      "from scipy.optimize import fsolve\n",
      "from pylab import *\n",
      "\n",
      "x1 = 1-(T1/T_c) \n",
      "def f12(P1):\n",
      "    return(math.log(P1/P_c)-(A*x1+B*x1**1.5+C*x1**3+D*x1**6)/(1-x1)) \n",
      "P1 = fsolve(f12,0.01) # [bar]\n",
      "print\"Vapor pressure of benzene at 320 K is\",round(P1[0],2),\"bar\"\n",
      "\n",
      "M_benzene = 78.0 # [gram/mole]\n",
      "print\"\\nSolution 8.1 (a)\"\n",
      "\n",
      "\t# Solution (a)\n",
      "\t# For nitrogen\n",
      "M_nitrogen = 28.0  \t\t\t\t# [gram/mole]\n",
      "\t# From equation 8.2\n",
      "Y = P1/(P_total - P1)  \t\t\t\t#[mole C6H6/ mole N2]\n",
      "Y_s1 = Y*(M_benzene/M_nitrogen)  \t\t# [gram C6H6/gram N2]\n",
      "\n",
      "#Result\n",
      "\n",
      "print\"Absolute humidity of mixture of benzene and nitrogen is\",round(Y_s1[0],2),\" gram C6H6/gram N2\\n\\n\"\n",
      "\n",
      "print\"\\nSolution 8.1 (b)\\n\"\n",
      "\t# Solution (b)\n",
      "\t# For carbon dioxide\n",
      "M_carbondioxide = 44.0  \t\t\t\t# [gram/mole]\n",
      "\t# From equation 8.2\n",
      "Y = P1/(P_total - P1)  \t\t\t\t#[mole C6H6/ mole C02]\n",
      "Y_s2 = Y*(M_benzene/M_carbondioxide)  \t\t# [gram C6H6/gram CO2]\n",
      "\n",
      "#Result\n",
      "\n",
      "print\"\\nAbsolute humidity of mixture of benzene and carbon dioxide is\",round(Y_s2[0],3),\"gram C6H6/gram CO2\\n\""
     ],
     "language": "python",
     "metadata": {},
     "outputs": [
      {
       "output_type": "stream",
       "stream": "stdout",
       "text": [
        "Vapor pressure of benzene at 320 K is 0.32 bar\n",
        "\n",
        "Solution 8.1 (a)\n",
        "Absolute humidity of mixture of benzene and nitrogen is 1.31  gram C6H6/gram N2\n",
        "\n",
        "\n",
        "\n",
        "Solution 8.1 (b)\n",
        "\n",
        "\n",
        "Absolute humidity of mixture of benzene and carbon dioxide is 0.833 gram C6H6/gram CO2\n",
        "\n"
       ]
      }
     ],
     "prompt_number": 1
    },
    {
     "cell_type": "heading",
     "level": 3,
     "metadata": {},
     "source": [
      "Example 8.2,Page number:480"
     ]
    },
    {
     "cell_type": "code",
     "collapsed": false,
     "input": [
      "#Variable declaration\n",
      "\t# A - water vapor    B - air\n",
      "\t# REference state is air\n",
      "\n",
      "T_ref = 273  \t\t\t\t\t# [Reference temperature, K]\n",
      "T = 303  \t\t\t\t\t# [K]\n",
      "P_total = 1  \t\t\t\t\t# [atm]\n",
      "P_A = 4.24  \t\t\t\t\t# [Vapor pressure of water at 303K, kPa]\n",
      "M_A = 18.0  \t\t\t\t\t# [gram/mole]\n",
      "M_B = 29.0  \t\t\t\t\t# [gram/mole]\n",
      "C_A = 1.884  \t\t\t\t\t# [kJ/kg.K]\n",
      "C_B = 1.005  \t\t\t\t\t# [kJ/kg.K]\n",
      "lamda = 2502.3  \t\t\t\t# [Latent heat of Vaporization at 273K, kJ/kg]\n",
      "\n",
      "#Calculation\n",
      "\n",
      "P_total = P_total*101.325  \t\t\t# [kPa]\n",
      "\n",
      "\t# From equation 8.2\n",
      "Y_s = P_A/(P_total - P_A)*(M_A/M_B)  \t\t#[kg H2O/ kg dry air]\n",
      "\n",
      "\n",
      "\n",
      "print\"Absolute humidity of mixture of water vapor and air is\",round(Y_s,3),\"kg H2O/kg dry air\" \n",
      "\t# From equation 8.3\n",
      "H_s = C_B*(T-T_ref) + Y_s*(C_A*(T-T_ref) + lamda)  \t# [kJ/kg dry air]\n",
      "\n",
      "#Result\n",
      "\n",
      "print\"Enthalpy per unit mass of dry air of a saturated mixture at 303 K and 1 atm is\",round(H_s,1),\"kJ/kg dry air\" \n",
      "print\"\\n\\nFollowing graph shows the result of similar calculations form 273 to 333 K\"\n",
      "x=[273,283,293,303,313,323,333]\n",
      "y=[9.48,29.36,57.57,99.75,166.79,275.58,461.50]\n",
      "a=plot(x,y)\n",
      "xlabel('$Temperature,K$')\n",
      "ylabel('$Hs,kJ/kg dry air$')\n",
      "show(a)"
     ],
     "language": "python",
     "metadata": {},
     "outputs": [
      {
       "output_type": "stream",
       "stream": "stdout",
       "text": [
        "Absolute humidity of mixture of water vapor and air is 0.027 kg H2O/kg dry air\n",
        "Enthalpy per unit mass of dry air of a saturated mixture at 303 K and 1 atm is 99.5 kJ/kg dry air\n",
        "\n",
        "\n",
        "Following graph shows the result of similar calculations form 273 to 333 K\n"
       ]
      },
      {
       "metadata": {},
       "output_type": "display_data",
       "png": "iVBORw0KGgoAAAANSUhEUgAAAZAAAAETCAYAAAAYm1C6AAAABHNCSVQICAgIfAhkiAAAAAlwSFlz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\n",
       "text": [
        "<matplotlib.figure.Figure at 0x768af60>"
       ]
      }
     ],
     "prompt_number": 2
    },
    {
     "cell_type": "heading",
     "level": 3,
     "metadata": {},
     "source": [
      "Example 8.3,Page number:482"
     ]
    },
    {
     "cell_type": "code",
     "collapsed": false,
     "input": [
      "#Variable declaration\n",
      "\t# A - water vapor   B - air\n",
      "\n",
      "T = 328  \t\t\t\t\t# [dry bulb temperature, K]\n",
      "P_total = 1.0  \t\t\t\t\t# [atm]\n",
      "H = 30.0  \t\t\t\t\t# [relative humidity, %]\n",
      "\n",
      "P_vapA = 15.73  \t\t\t\t# [vapor pressure of water, kPa]\n",
      "P_total = P_total*101.325  \t\t\t# [kPa]\n",
      "M_A = 18.0  \t\t\t\t\t# [gram/mole]\n",
      "M_B = 29.0  \t\t\t\t\t# [gram/mole]\n",
      "\n",
      "P_A = (H/100)*P_vapA \t\t\t\t# [partial pressure of A,kPa]\n",
      "\n",
      "#Calculation\n",
      "\n",
      "\n",
      "print\"Solution 8.3 (a)\"\n",
      "\t# At dew point partial pressure is equal to vapor pressure\n",
      "\t# Using Antonnie equation we can find dew point temperature\n",
      "\n",
      "\n",
      "\n",
      "print\"Dew point temperature is 304.5 K\\n\"\n",
      "\n",
      "\t# From equation 8.1\n",
      "Y_s = P_A/(P_total-P_A)*(M_A/M_B) \n",
      "\n",
      "\n",
      "print\"Absolute humidity of air-water mixture at 328 K is\",round(Y_s,2),\"kg H2O/kg dry air\\n\\n\"\n",
      "\n",
      "print\"\\n Solution8.3 (b)\"\n",
      "\n",
      "\t#soluton (b)\n",
      "T_ref = 273  \t\t\t\t\t# [K]\n",
      "C_A = 1.884  \t\t\t\t\t# [kJ/kg.K]\n",
      "C_B = 1.005  \t\t\t\t\t# [kJ/kg.K]\n",
      "lamda = 2502.3  \t\t\t\t# [Latent heat of Vaporization at 273 K, kJ/kg]\n",
      "\n",
      "\t# From equation 8.3\n",
      "H_s = C_B*(T-T_ref) + Y_s*(C_A*(T-T_ref) + lamda) \n",
      "\n",
      "\n",
      "print\"Enthalpy per unit mass of dry air of a saturated mixture relative to 273 K is\",round(H_s,1),\"kJ/kg dry air\""
     ],
     "language": "python",
     "metadata": {},
     "outputs": [
      {
       "output_type": "stream",
       "stream": "stdout",
       "text": [
        "Solution 8.3 (a)\n",
        "Dew point temperature is 304.5 K\n",
        "\n",
        "Absolute humidity of air-water mixture at 328 K is 0.03 kg H2O/kg dry air\n",
        "\n",
        "\n",
        "\n",
        " Solution8.3 (b)\n",
        "Enthalpy per unit mass of dry air of a saturated mixture relative to 273 K is 134.3 kJ/kg dry air\n"
       ]
      }
     ],
     "prompt_number": 3
    },
    {
     "cell_type": "heading",
     "level": 3,
     "metadata": {},
     "source": [
      "Example 8.4,Page number:483"
     ]
    },
    {
     "cell_type": "code",
     "collapsed": false,
     "input": [
      "#Variable declaration\n",
      "\n",
      "\t# a - water vapor    b - air\n",
      "T_G1 = 356  \t\t\t\t\t\t# [K]\n",
      "P_total = 101.325  \t\t\t\t\t# [kPa]\n",
      "Y_1 = .03  \t\t\t\t\t\t# [kg water/kg dry air]\n",
      "\n",
      "\n",
      "C_pa = 1.884 \t \t\t\t\t\t# [kJ/kg.K]\n",
      "C_pb = 1.005  \t\t\t\t\t\t# [kJ/kg.K]\n",
      "\n",
      "C_s1 = C_pb + Y_1*C_pa \t\t\t\t\t# [kJ/kg.K]\n",
      "\n",
      "T_1 = 373.15  \t\t\t\t\t\t# [K]\n",
      "T_c = 647.1  \t\t\t\t\t\t# [K]\n",
      "M_a = 18.02  \t\t\t\t\t\t# [gram/mole]\n",
      "M_b = 28.97  \t\t\t\t\t\t# [gram/mole]\n",
      "lamda_1 = 2256  \t\t\t\t\t# [Latent Heat of Vaporizarion at T_1, \n",
      "#Calculation\n",
      "\t\t\t\t\t\t\t\n",
      "import math\n",
      "from scipy.optimize import fsolve\n",
      "from pylab import *\n",
      "\n",
      "def f12(T_as):\n",
      "    return(T_as - T_G1 + ((math.exp(16.3872 - (3885.7/(T_as - 42.98)))/(P_total - (math.exp(16.3872 - (3885.7/(T_as - 42.98))))))*(M_a/M_b) - Y_1)*(lamda_1*((1-T_as/T_c)/(1-T_1/T_c))**.38/C_s1)) \n",
      "T_as = fsolve(f12,310)  # [K]\n",
      "\n",
      "\n",
      "print\"Adiabatic Saturation Temperature is\",round(T_as[0]),\"K\"\n",
      "\n",
      "\t# Now using equation 8.2\n",
      " \n",
      "P_a = math.exp(16.3872-(3885.7/(T_as-42.98)))  \t\t# [kPa]\n",
      "Y_as = P_a/(P_total-P_a)*M_a/M_b  \t\t\t# [kg water/kg dry air]\n",
      "\n",
      "#Result\n",
      "print\"Absolute humidity is\",round(Y_as,3),\"kg water/kg dry air\\n\""
     ],
     "language": "python",
     "metadata": {},
     "outputs": [
      {
       "output_type": "stream",
       "stream": "stdout",
       "text": [
        "Adiabatic Saturation Temperature is 313.0 K\n",
        "Absolute humidity is 0.049 kg water/kg dry air\n",
        "\n"
       ]
      }
     ],
     "prompt_number": 4
    },
    {
     "cell_type": "heading",
     "level": 3,
     "metadata": {},
     "source": [
      "Example 8.5,Page number:487"
     ]
    },
    {
     "cell_type": "code",
     "collapsed": false,
     "input": [
      "#Variable declaration\n",
      "\n",
      "T_w = 320  \t\t\t\t\t\t# [K]\n",
      "T_g = 340  \t\t\t\t\t\t# [K]\n",
      "lambda_w = 2413  \t\t\t\t\t# [Latent Heat of Vaporization at 320K, \t\t\t\t\t\t\tkJ/kg]\n",
      "Y_w1 = 0.073  \t\t\t\t\t\t# [kg water/kg dry air]\n",
      "\n",
      "A = 0.95  \t\t\t\t\t\t# [For air water system,A, kJ/kg.K]\n",
      "\n",
      "#Calculation\n",
      "\n",
      "\t#    here A = hg/ky, psychrometric ratio\n",
      "\t#    Air-water mixture is saturated at 320K and 1 atm\n",
      "\t#    Using equation 8.15\n",
      " \n",
      "Y_w2 = Y_w1 - ((T_g-T_w)*A/lambda_w)  \t\t\t# [kg water/kg dry air]\n",
      "\n",
      "#Result\n",
      "\n",
      "print\"Absolute humidity of air-water mixture at 340 K and 1 atm is\",round(Y_w2,3),\" kg water/kg dry air\\n \""
     ],
     "language": "python",
     "metadata": {},
     "outputs": [
      {
       "output_type": "stream",
       "stream": "stdout",
       "text": [
        "Absolute humidity of air-water mixture at 340 K and 1 atm is 0.065  kg water/kg dry air\n",
        " \n"
       ]
      }
     ],
     "prompt_number": 5
    },
    {
     "cell_type": "heading",
     "level": 3,
     "metadata": {},
     "source": [
      "Example 8.6,Page number:487"
     ]
    },
    {
     "cell_type": "code",
     "collapsed": false,
     "input": [
      "\n",
      "#Variable declaration\n",
      "\t# a - toluene    b - air\n",
      "\n",
      "T_G1 = 333  \t\t\t\t\t\t# [K]\n",
      "P_total = 101.325  \t\t\t\t\t# [kPa]\n",
      "Y_1 = 0.05  \t\t\t\t\t\t# [kg vapor/kg dry air]\n",
      "\n",
      "\n",
      "C_pa = 1.256  \t\t\t\t\t\t# [kJ/kg.K]\n",
      "C_pb = 1.005  \t\t\t\t\t\t# [kJ/kg.K]\n",
      "\n",
      "C_s1 = C_pb + Y_1*C_pa\n",
      "\n",
      "T_1 = 383.8  \t\t\t\t\t\t# [K]\n",
      "T_c = 591.8  \t\t\t\t\t\t# [K]\n",
      "M_a = 92  \t\t\t\t\t\t# [gram/mole]\n",
      "M_b = 28.97  \t\t\t\t\t\t# [gram/mole]\n",
      "\n",
      "#Calculation\n",
      "lamda_1 = 33.18*1000/92  \t\t\t\t# [Latent heat of vaporization at T_1, \t\t\t\t\t\t\tkJ/kg]\n",
      "import math\n",
      "from scipy.optimize import fsolve\n",
      "\t# Constants of antoine equation\n",
      "A = 13.9320 \n",
      "B = 3057  \t\t\t\t\t\t# [K]\n",
      "C = -55.52  \t\t\t\t\t\t# [K]\n",
      "\n",
      "print \"\\nSolution 8.6 (a)\"\n",
      "\n",
      "\t# Solution (a)\n",
      "\n",
      "def f12(T_as):\n",
      "    return(T_as - T_G1 + ((math.exp(13.9320 - (3057/(T_as - 55.52)))/(P_total - (math.exp(13.9320 - (3057/(T_as - 55.52))))))*(M_a/M_b) - Y_1)*(lamda_1*((1-T_as/T_c)/(1-T_1/T_c))**0.38/C_s1))\n",
      "T_as = fsolve(f12,273)   \t\t\t\t# [K]     \n",
      "print\"Adiabatic Saturation Temperature is\",round(T_as),\"K\"\n",
      "\n",
      "\t# Now using equation 8.2\n",
      " \n",
      "P_a = math.exp(13.9320-(3057/(T_as-55.52)))  \t\t# [kPa]\n",
      "Y_as = P_a/(P_total-P_a)*M_a/M_b  \t\t\t# [kg vapor/kg dry air]\n",
      "\n",
      "#Result\n",
      "\n",
      "print\"Absolute humidity is\",round(Y_as,3),\"kg vapor/kg dry air\"\n",
      "\n",
      "\n",
      "\n",
      "print\"\\nSolution 8.6 (b)\"\n",
      "\n",
      "# Solution (b)\n",
      "\n",
      "# Thermodynamic properties of mixture of toluene and air\n",
      "row = 1.06  \t\t\t\t\t\t# [kg/cubic m]\n",
      "u = 19.5*10**-6  \t\t\t\t\t# [P]\n",
      "Pr = 0.7 \n",
      "Dab = 0.1  \t\t\t\t\t\t#[From Wilke-Lee equation, square cm/s]\n",
      "Sc = u/(row*Dab*10**-4) \n",
      "\n",
      "# Using equation 8.16\n",
      "\n",
      "A_1 = C_s1*(Sc/Pr)**0.567  # [kJ/kg.K]\n",
      "\t# here A_1 = hg/ky, psychrometric ratio\n",
      "\n",
      "\t# Using equation 8.15\n",
      "\t#    T_w = T_G1 - (Y_w-Y_1)*lamda_w/(hg/ky)\n",
      "\t#   where lamda_w = lamda_1*((1-T_w/T_c)/(1-T_1/T_c))**.38\n",
      "\t#   Y_w = P_a/(P_total-P_a)*M_a/M_b\n",
      "\t#   P_a = math.exp(A-B/(T+c))\n",
      "\n",
      "def f15(T_w):\n",
      "    return(T_w - T_G1 + ((math.exp(13.9320 - (3057/(T_w - 55.52)))/(P_total - (math.exp(13.9320 - (3057/(T_w - 55.52))))))*(M_a/M_b) - Y_1)*(lamda_1*((1-T_w/T_c)/(1-T_1/T_c))**.38/A_1)) \n",
      "T_w = fsolve(f15,273)  # [K]\n",
      "print\"Wet bulb Temperature is\",round(T_w),\" K\\n\"\n",
      "\n",
      "# Now using equation 8.2\n",
      " \n",
      "P_a = math.exp(13.9320-(3057/(T_w-55.52)))  \t\t# [kPa]\n",
      "Y_w = P_a/(P_total-P_a)*M_a/M_b  \t\t\t# [kg vapor/kg dry air]\n",
      "\n",
      "print\"Absolute humidity is\",round(Y_w,3),\"kg vapor/kg dry air\\n\"\n"
     ],
     "language": "python",
     "metadata": {},
     "outputs": [
      {
       "output_type": "stream",
       "stream": "stdout",
       "text": [
        "\n",
        "Solution 8.6 (a)\n",
        "Adiabatic Saturation Temperature is 300.0 K\n",
        "Absolute humidity is 0.136 kg vapor/kg dry air\n",
        "\n",
        "Solution 8.6 (b)\n",
        "Wet bulb Temperature is 305.0  K\n",
        "\n",
        "Absolute humidity is 0.177 kg vapor/kg dry air\n",
        "\n"
       ]
      }
     ],
     "prompt_number": 6
    },
    {
     "cell_type": "heading",
     "level": 3,
     "metadata": {},
     "source": [
      "Example 8.7,Page number:493"
     ]
    },
    {
     "cell_type": "code",
     "collapsed": false,
     "input": [
      "L_min = 2.27 # [kg/square m.s]\n",
      "G_min = 2 # [kg/square m.s]\n",
      "L2_prime = 15 # [kg/s]\n",
      "Templ2 = 318 # [K]\n",
      "Tempg1 = 303 # [Entering air dry bulb, K]\n",
      "Tempw1 = 297 # [ Entering air wet bulb, K]\n",
      "Kya = 0.90 # [kg/cubic m.s]\n",
      "\n",
      "import math\n",
      "from pylab import *\n",
      "from scipy.optimize import fsolve\n",
      "from numpy import*\n",
      "\n",
      "H1_prime = 72.5 # [kJ/kg dry air]\n",
      "Y1_prime = 0.0190 # [kg water/kg dry air]\n",
      "Templ1 = 302 # [K]\n",
      "Cal = 4.187 # [kJ/kg]\n",
      "\n",
      "# Equilibrium Data:\n",
      "# Data  = [Temp.(K),H_star(kJ/kg)]\n",
      "Data_star =matrix([[302,100],[305.5,114],[308,129.8],[310.5,147],[313,166.8],[315.5,191],[318,216]]) \n",
      "\n",
      "# The operating line for least slope:\n",
      "H2_star = 210 # [kJ/kg]\n",
      "Data_minSlope =matrix([[Templ1,H1_prime],[Templ2,H2_star]]) \n",
      "def f14(Gmin):\n",
      "    return(((L2_prime*Cal)/Gmin)-((H2_star-H1_prime)/(Templ2-Templ1))) \n",
      "Gmin = fsolve(f14,2) # [kg/s]\n",
      "Gs = 1.5*Gmin # [kg/s]\n",
      "\n",
      "# For the Operating Line:\n",
      "def f15(H2):\n",
      "    return(((H2-H1_prime)/(Templ2-Templ1))-((L2_prime*Cal)/Gs)) \n",
      "H2 = fsolve(f15,2) # [kJ/kg dry air]\n",
      "Data_opline =matrix([[Templ1,H1_prime],[Templ2,H2]]) \n",
      "\n",
      "\n",
      "a1=plot(Data_star[:,0],Data_star[:,1],label='$Equilibrium line$')\n",
      "a2=plot(Data_minSlope[:,0],Data_minSlope[:,1],label='$Minimum Flow Rate Line$')\n",
      "a3=plot(Data_opline[:,0],Data_opline[:,1],label='$Operating Line$') \n",
      "legend(loc='upper right')\n",
      "title('Operating Diagram')\n",
      "xlabel(\"$Liquid Temperature, K$\") \n",
      "ylabel(\"$Enthalphy Of Air Water vapour, kJ / kg dry air$\") \n",
      "show(a1)\n",
      "show(a2)\n",
      "show(a3)\n",
      "\n",
      "# Tower cross section Area:\n",
      "Al = L2_prime/L_min # [square m]\n",
      "Ag = Gs/G_min # [square m]\n",
      "A = min(Al,Ag) # [square m]\n",
      "print\"Cross sectional is\",round(A[0],2),\" square m\\n\"\n",
      "\n",
      "# Data from operating line:\n",
      "# Data1 = [Temp.(K),H_prime(kJ/kg)]\n",
      "Data1 =matrix([[302,72.5],[305.5,92],[308,106.5],[310.5,121],[313,135.5],[315.5,149.5],[318,164.2]]) \n",
      "\n",
      "# Driving Force:\n",
      "Data2 = zeros((7,2)) \n",
      "# Data2 = [Temp[K],driving Force]\n",
      "for i in range(0,7):\n",
      "    Data2[i][0] = Data1[i,0] \n",
      "    Data2[i,1] = 1/(Data_star[i,1]-Data1[i,1]) \n",
      "\n",
      "\n",
      "# The data for operating line as abcissa is plotted against driving force \n",
      "Area = 3.28 \n",
      "# From Eqn. 7.54\n",
      "def f16(Z):\n",
      "    return(Area-(Kya*Z/G_min)) \n",
      "Z = fsolve(f16,2) \n",
      "print\"The height of tower is\",round(Z[0],2)\n",
      "NtoG = 3.28 \n",
      "HtoG = G_min/Kya # [m]\n",
      "\n",
      "# Make up water\n",
      "# Assuming the outlet air is essentially saturated:\n",
      "Y2_prime = 0.048 # [kg water/kg dry air]\n",
      "H2 = 164.2  # [kJ/kg dry air]\n",
      "# This corresponds to an exit-air temperature of 312.8 K\n",
      "\n",
      "# Approximate rate of evaporation \n",
      "R = Gs*(Y2_prime-Y1_prime) \n",
      "print\"Rate of evaporation is\",round(R[0],3),\"kg/s\\n\""
     ],
     "language": "python",
     "metadata": {},
     "outputs": [
      {
       "metadata": {},
       "output_type": "display_data",
       "png": "iVBORw0KGgoAAAANSUhEUgAAAZAAAAEdCAYAAAAikTHKAAAABHNCSVQICAgIfAhkiAAAAAlwSFlz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\ny/8DTPMaHFV/PpQAAAAASUVORK5CYII=\n",
       "text": [
        "<matplotlib.figure.Figure at 0x7e784a8>"
       ]
      },
      {
       "output_type": "stream",
       "stream": "stdout",
       "text": [
        "Cross sectional is 5.48  square m\n",
        "\n",
        "The height of tower is 7.29\n",
        "Rate of evaporation is 0.318 kg/s\n",
        "\n"
       ]
      }
     ],
     "prompt_number": 2
    },
    {
     "cell_type": "heading",
     "level": 3,
     "metadata": {},
     "source": [
      "Example 8.8,Page number:495"
     ]
    },
    {
     "cell_type": "code",
     "collapsed": false,
     "input": [
      "\n",
      "print\"Solution8.8 (a)\"\n",
      "\n",
      "#Variable declaration\n",
      "\t# a - water vapor   b - air\n",
      "\n",
      "T_L2 = 314  \t\t\t\t# [inlet water temperature, K]\n",
      "T_L1 = 303  \t\t\t\t# [outlet water temperature, K]\n",
      "T_d = 306  \t\t\t\t# [dry bulb temperature ,K]\n",
      "T_w1 = 298  \t\t\t\t# [wet bulb temperature, K]\n",
      "Z = 3 \t\t \t\t\t# [packed tower depth, m]\n",
      "G_x = 3  \t\t\t\t# [mass velocity, kg/square m.s]\n",
      "G_s =2.7  \t\t\t\t# [mass velocity, kg/square m.s]\n",
      "\n",
      "import math\n",
      "from numpy import *\n",
      "from pylab import *\n",
      "T_o = 273  \t\t\t\t# [reference temperature, K]\n",
      "C_al = 4.187  \t\t\t\t# [kJ/kg.K]\n",
      "C_pb = 1.005  \t\t\t\t# [kJ/kg.K]\n",
      "C_pa = 1.884  \t\t\t\t# [kJ/kg.K]\n",
      "P_total = 101.325  \t\t\t# [kPa]\n",
      "lamda_0 = 2502.3  \t\t\t# [kJ/kg]\n",
      "M_a = 18.02  \t\t\t\t# [gram/mole]\n",
      "M_b = 28.97  \t\t\t\t# [gram/mole]\n",
      "\n",
      "\t# Equilibrium Data:\n",
      "\t# Data  = [Temp.(K),H_eqm(kJ/kg)],[H_eqm - Equilibrium gas enthalpy]\n",
      "Data_eqm =matrix([[273,9.48],[283,29.36],[293,57.8],[303,99.75],[313,166.79],[323,275.58],[333,461.5]]) \n",
      "\n",
      "\n",
      "a1=plot(Data_eqm[:,0],Data_eqm[:,1],label='$Equilibrium line$') \n",
      "\n",
      "legend(loc='upper right') \n",
      "xlabel(\"Liquid Temperature, K\") \n",
      "ylabel(\"Enthalphy Of Air Water vapour, kJ / kg dry air\") \n",
      "\n",
      "P_a = math.exp(16.3872-(3885.7/(T_w1-42.98)))  \t\t\t\t# [kPa]\n",
      "Y_m1 = P_a/(P_total-P_a)*(M_a/M_b)  \t\t\t\t\t# [kg water/kg dry air]\n",
      "H_g1 = C_pb*(T_w1-T_o) + Y_m1*(C_pa*(T_w1-T_o)+lamda_0)  \t\t# [Enthalpy of saturated \t\t\t\t\t\t\t\t\tmixture, kJ/kg \t\t\t\t\t\t\t\t\tdry air]\n",
      "\n",
      "\t# From overall energy balance\n",
      "H_g2 = H_g1 + G_x*C_al*(T_L2-T_L1)/G_s  \t\t\t\t# [Enthalpy of exit air, \t\t\t\t\t\t\t\t\tkJ/kg]\n",
      "\n",
      "\t\t# For calculation of mass transfer unit, Ntog\n",
      "\t\t# Data1 = [T_L1 H_g1,.....,T_L2 H_g2]\n",
      "\n",
      "deltaT = (T_L2-T_L1)/9 \n",
      "\n",
      "\t# Data for enthalpy of exit air at different temperature varying from T_L1 to T_L2, \t\t\toperating line\n",
      "Data1=matrix([[303,76.17],[304.22,81.85],[305.44,87.53],[306.67,93.22],[307.89,98.91],[309.11,104.59],[310.33,110.28],[311.56,115.96],[312.78,121.65],[314,127.35]]) \n",
      "\n",
      "\t# Data of equilibrium gas enthalpy at different temperature varying from T_L1 to T_L2 \t\tfrom the above equilibrium graph \n",
      "\n",
      "Data2=matrix([[303,100],[304.22,107.93],[305.44,116.12],[306.67,124.35],[307.89,132.54],[309.11,140.71],[310.33,148.89],[311.56,157.14],[312.78,165.31],[314,177.67]]) \n",
      "\n",
      "\t# Driving force \n",
      "Data3 = zeros((10,3)) \n",
      "\t# Data3 =[Equilibrium gas enthalpy, driving force]\n",
      "for i in range(0,10):\n",
      "    Data3[i][0] = Data1[i,1] \n",
      "    Data3[i][1] = 1/(Data2[i,1]-Data1[i,1]) \n",
      "\n",
      "\n",
      "\t# The data for Equilibrium gas enthalpy as abcissa is plotted against driving force\n",
      "Area = 1.642 \n",
      "N_tog = 1.642 \n",
      "H_tog = Z/N_tog  \t\t\t\t# [m]\n",
      "\n",
      "\t# Overall volumetric mass-transfer coefficient, K_ya\n",
      "K_ya = G_s/H_tog \n",
      "print\"Overall volumetric mass-transfer coefficient is\",K_ya,\"kg/cubic m.s\\n\\n\" \n",
      "\n",
      "print\"\\nSolution (b)\"\n",
      "#Illustration 8.8 (b) \n",
      "\n",
      "T_w2 = 288  \t\t\t\t\t# [New entering-air wet-bulb temperature, K]\n",
      "P_a2 = math.exp(16.3872-(3885.7/(T_w2-42.98)))  # [kPa]\n",
      "Y_m2 = P_a2/(P_total-P_a2)*(M_a/M_b)  \t\t# [kg water/kg dry air]\n",
      "H_g11 = C_pb*(T_w2-T_o) + Y_m2*(C_pa*(T_w2-T_o)+lamda_0)  \t# [Enthalpy of saturated mixture, \t\t\t\t\t\t\t\tkJ/kg dry air]\n",
      "\n",
      "\t# the change in water temperature through the tower must remain the same as in part (a), \t\tnamely T_L2b-T_L1b = 11K \n",
      "\t# Since N_tog is a function of both water temperatures(T_L1',T_L2'), this provides the \t\t\tsecond relation needed to calculate the values of T_L2b and T_L1b\n",
      "\t# The two equations are solved simultaneously by trial and error method, from above we \t\tget T_L1'= 297K\n",
      "T_L1b = 297  \t\t\t\t\t# [K]\n",
      "T_L2b = T_L1b + 11  \t\t\t\t#[K]\n",
      "S =  T_L1b - T_w2  \t\t\t\t# [wet bulb temperature approach, K]\n",
      "\n",
      "#Result\n",
      "\n",
      "print\"The outlet water temperature and wet bulb temperature approach is\",T_L1b,\" K and \",S,\" K respectively \"\n"
     ],
     "language": "python",
     "metadata": {},
     "outputs": [
      {
       "output_type": "stream",
       "stream": "stdout",
       "text": [
        "Solution8.8 (a)\n",
        "Overall volumetric mass-transfer coefficient is"
       ]
      },
      {
       "output_type": "stream",
       "stream": "stdout",
       "text": [
        " 1.4778 kg/cubic m.s\n",
        "\n",
        "\n",
        "\n",
        "Solution (b)\n",
        "The outlet water temperature and wet bulb temperature approach is 297  K and  9  K respectively \n"
       ]
      },
      {
       "metadata": {},
       "output_type": "display_data",
       "png": "iVBORw0KGgoAAAANSUhEUgAAAYoAAAEPCAYAAABcA4N7AAAABHNCSVQICAgIfAhkiAAAAAlwSFlz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\n",
       "text": [
        "<matplotlib.figure.Figure at 0x7cb6240>"
       ]
      }
     ],
     "prompt_number": 8
    }
   ],
   "metadata": {}
  }
 ]
}